0 Daumen
339 Aufrufe

Aufgabe:


Untersuchen Sie die Folge auf Konvergenz bzw. Divergenz. Bestimmen Sie ggf. den Grenzwert.


\( \frac{n+2}{2^n -n} \)


Problem/Ansatz:

Ich vermute, dass die Folge gegen 0 konvergiert.

Ich möchte also zeigen: Für jedes ε>0 existiert ein N∈ℕ, so dass I \( \frac{n+2}{2^n -n} \) -0 I <ε für alle n≥N.

Ich fange an mit:

Sei ε>0 beliebig.

Und ich muss irgendwie so abschätzen, dass ich von I \( \frac{n+2}{2^n -n} \) - 0 I = I \( \frac{n+2}{2^n -n} \) I auf ... <ε komme.

Leider weiß ich generell nicht so richtig, wie ich vorgehen soll oder was ich jetzt machen darf. Ist es sinnvoll, erstmal die Betragsstriche wegzulassen? Oder darf ich das gar nicht?


Danke für jede Hilfe!

Avatar von

Kannst Betragsstriche entfernen, da der Bruch für alle n größer gleich 1 nichtnegativ ist.

Ok, danke!


Meine nächste Idee wäre dann, mit 2^n zu erweitern, wodurch man dann den Ausdruck

\( \frac{\frac{n}{2^n}+\frac{2}{2^n}}{1-\frac{n}{2^n}} \) hätte. Und dann vielleicht abschätzen, dass all diese Teilbrüche kleinergleich \( \frac{1}{n} \) sind.


Also dass der ganze Ausdruck ≤ \( \frac{\frac{1}{n}+\frac{1}{n}}{1-\frac{1}{n}} \) ist.

Du machst es dir ziemlich schwer. Du könntest doch das Folgenglied nach oben durch eine Nullfolge abschätzen.

Zum Beispiel gilt für \(n\geq 2\):

\(2^n = (1+1)^n \geq 1+n+\frac{n(n-1)}2\)

Damit bekommst du für \(n\geq 2\):

 \(2^n-n > \frac{n(n-1)}2\)

Danke für deine Antwort. Ich werde versuchen, deinen Ansatz nachzuvollziehen :)

Ich kenne halt noch keine Tricks, deswegen "mache ich es mir schwer", weil es das einzige ist, was ich bisher im Repertoire habe. Im Skript stand halt, dass man abschätzen kann \( \frac{n}{2^n} \)≤\( \frac{1}{n} \) für n>3.

Ich verstehe nicht so richtig, was du mit "nach oben durch eine Nullfolge abschätzen" meinst. Was ist denn genau in deinem Ansatz die Nullfolge? Ich kenne bereits 1/n als Nullfolge.

Eine etwas andere Methode: Bekanntlich gilt \(2^n\ge n^2\) für alle \(n\ge4\). Damit ist$$\lvert a_n\rvert=\left\lvert\frac{n+2}{2^n-n}\right\rvert=\frac{n+2}{2^n-n}<\frac{n+2}{n^2-n}+\frac2n=\frac3{n-1}.$$

Wow, cool, danke, das kann ich gut nachvollziehen und benutzen :)

1 Antwort

0 Daumen

Ich setze mal meinen obigen Kommentar hier als Antwort fort.

Man kann deine Folge "einschließen" mit obigem Trick. Für \(n\geq 2\) hast du

$$0\leq \frac{n+2}{2^n -n} \stackrel{2^n -n >\frac{n(n-1)}{2} }{<}2\frac{n+2}{n(n-1)}=\frac 2{n-1}+\frac 4{n(n-1)}< \frac 6{n-1}$$

Nun kannst du das natürlich für deine \(\varepsilon\)-Akrobatik benutzen oder du zitierst den Einschließungssatz, laut dem nun deine Folge konvergent gegen 0 sein muss.

Avatar von 10 k

Vielen Dank für deine Mühe!


Leider kann ich deinen Schritt mit (1+1)^n ≥ ... noch nicht nachvollziehen. Kannst du mir da vielleicht einen Tipp geben?

Es gibt die berühmten Binomischen Formeln (siehe Abschnitt "Höhere Potenzen ..."). Das ist ein oft und gern genutztes Tool.

Alles klar, mit so hohen Potenzen kannte ich sie nicht.

Werd ich mir merken!

Warum genügt es nicht mit 2^n zu kürzen oder zu sagen, dass 2^n als stärkster Term

gewinnt? Die Nullfolge ist doch offensichtlich.

2^n macht alles andere "platt".

Ich werde mal meinen Übungsleiter fragen, ob wir schon so argumentieren dürfen. Es ist unser erstes Übungsblatt, auf dem wir einen solchen Konvergenzbeweis haben und ich glaube, sie wollen es noch nicht so salopp

2n macht alles andere "platt".

Bringt nur ohne Nachweis nichts. Es ist nicht sinnvoll, Mathematikanfängern gleich die eleganteste Lösung zu präsentieren. Gerade was das Abschätzen solcher Terme angeht, sollte man das ruhig mal üben, um auch ein Gefühl zu bekommen, wie es klappen könnte.

Ein anderes Problem?

Stell deine Frage

Ähnliche Fragen

0 Daumen
1 Antwort
0 Daumen
1 Antwort
0 Daumen
1 Antwort
0 Daumen
1 Antwort

Willkommen bei der Mathelounge! Stell deine Frage einfach und kostenlos

x
Made by a lovely community